MOC Questions

Publication
Article
Cardiology Review® OnlineJune 2007
Volume 24
Issue 6

Preparing for the American Board of Internal Medicine Maintenance of Certification

The following questions can assist candidates for the Maintenance of Certification Exam in Cardiovascular Disease prepare for this test. We hope you find this helpful and welcome your feedback.

Questions

What is the most appropriate next step?

1. A 64-year-old woman is evaluated in the intensive care unit 2 days after an acute anterior wall myocardial infarction (MI). She was treated with primary stent implantation in the mid-left anterior descending (LAD) artery. When seen today, she is clinically asymptomatic but has bibasilar rales on examination. Her blood pressure is 124/68 mm Hg, heart rate 66 beats per minute (bpm), and respiratory rate 18 breaths per minute (bpm). Echocardiography shows anterior hypokinesis and a left ventricular ejection fraction (LVEF) of 23%. Her medications include atorvastatin, aspirin, clopidogrel, a beta blocker, and an angiotensin-converting enzyme (ACE)-inhibitor.

  1. Repeat coronary angiography.
  2. Addition of an angiotensin receptor blocker.
  3. Addition of amiodarone.
  4. Addition of eplerenone.
  5. Submaximal stress testing.

What is the next step in management?

2. A 50-year-old asymptomatic man is evaluated during a routine follow-up visit for heart failure. A stress test at the time of initial diagnosis 2 years ago was negative for ischemia. Currently, he jogs 2 miles each morning without dyspnea, palpitations, or chest pain. An echocardiogram performed this morning showed mild mitral regurgitation and left ventricular enlargement (LVIDd 6.2 cm) with a LVEF of 38%. His current medications include lisinopril and hydrochlorothiazide. On physical examination, his heart rate is 72 bpm, blood pressure 135/80 mm Hg, and respiratory rate 14 bpm. His clinical examination is otherwise unremarkable.

  1. Repeat exercise nuclear perfusion study in 1 year.
  2. Addition of spironolactone.
  3. Addition of candasartan.
  4. Addition of digoxin.
  5. Addition of carvedilol.

What would be the most appropriate next test to perform?

3. A 67-year-old man with chronic knee and hip arthritis presents with worsening right lower extremity edema, dyspnea, fatigue, and anorexia over the past 4 months. Despite apparent weight loss, the patient has in fact gained 7-kg (15.4 lb). On examination, he has a heart rate of 97 bpm, a blood pressure of 110/82 mm Hg in both arms, a respiratory rate of 22 bpm, and an oxygen saturation of 92%. Jugular veins are distended and increase during sustained inspiration. His lungs are clear. He has an S4 and a I/VI holosystolic murmur that increases with inspiration. His liver is detected 3 cm below the right costal margin. He has 2+ pitting edema at the knees. An electrocardiogram (ECG) reveals low limb voltage without evidence of resting ischemia. His B-type natriuretic peptide (BNP) level is 826 pg/mL. An echocardiogram reveals a small pericardial effusion with moderate biatrial enlargement, septal and posterior free-wall thickening (2.0 cm each), and a small left ventricular cavity. No regional wall motion abnormalities are reported and the LVEF is 56%.

  1. Workup for occult secondary hypertension.
  2. Cardiac angiography.
  3. Simultaneous right and left heart catheterization.
  4. Abdominal fat pad biopsy.
  5. Pharmacologic nuclear perfusion testing.

What should be done next?

4. A 55-year-old woman is seen by you 4 years after an MI after having been told to lower her dose of lisinopril last week by her internist. Her last echocardiogram revealed inferolateral hypokinesis with a LVEF of 38%. She notes increased fatigue but is otherwise asymptomatic. Her medications include carvedilol, lisinopril, atorvastatin, isosorbide mononitrate, and aspirin. On examination, her heart rate is 60 bpm and regular, blood pressure is 145/75 mm Hg, and respiratory rate is 14 bpm. Her examination is otherwise unremarkable. Recent laboratory analyses reveal potassium 5.9 meq/L (5.9 mmol/L), bicarbonate 24 meq/L (24 mmol/L), blood urea nitrogen 45 mg/dL (16.1 mmol/L), and creatinine 2.1 (185.6 µmol/L).

  1. Discontinue lisinopril, begin hydralazine.
  2. Discontinue lisinopril, begin candasartan.
  3. Discontinue lisinopril, begin spironolactone.
  4. Renal duplex ultrasonography to assess renal artery blood flow.
  5. Encourage oral hydration.

x

y

What is the most appropriate next step?

5. A 46-year-old woman comes to you complaining of increasing fatigue, exertional dyspnea, and ankle edema over the past 8 months. Her medical history is unremarkable except for being successfully treated for Hodgkin’s disease 20 years ago with chemotherapy and radiation. Her heart rate is 96 bpm, respiratory rate 22 bpm, and blood pressure is 97/60 mm Hg. She has jugular venous distention to the angle of the jaw with prominent and descents. She has no pulsus paradoxus and Kussmaul’s sign is absent. Her lungs are clear to auscultation bilaterally. She has an early diastolic sound. She also has right upper quadrant tenderness and 3+ bilateral lower-extremity edema. Her ECG reveals only a sinus tachycardia. A chest radiograph is unremarkable. Echocardiography shows normal wall thickness, moderate left atrial enlargement with mild mitral regurgitation, no wall motion abnormalities, and an LVEF of 55%. Her right ventricular systolic pressure is 52 mm Hg, and no pericardial effusion is present. During inspiration, there is a 52% increase in right ventricular filling.

  1. Transesophageal echocardiography.
  2. Cardiac computed tomography (CT) or magnetic resonance imaging (MRI).
  3. Coronary angiography and myocardial biopsy.
  4. Dobutamine echocardiography.
  5. Right upper quadrant ultrasonography.

You should recommend:

6. You consult on a 69-year-old man who is about to undergo carotid endarterectomy for a severe left internal carotid artery stenosis in the setting of a 1-week history of transient right arm weakness. Prior to the event, he reports slowly jogging 1 mile a day, 4 days a week without symptoms. However, he reports rare episodes of momentary chest pain while reading in bed late at night. He quit smoking 22 years ago and has no other cardiac risk factors. On physical examination, his neck veins are hard to appreciate. You hear no carotid bruits or murmurs nor do you appreciate either an S3 or S4. He has coarse breath sounds bilaterally.

  1. He be given a beta blocker and taken to surgery.
  2. He be given a beta blocker, aspirin, and an exercise nuclear perfusion or dobutamine stress echocardiographic study.
  3. He should be given an exercise nuclear perfusion dobutamine stress echocardiographic study off medications.
  4. He should undergo coronary angiography before surgery.
  5. He should be placed on warfarin, aspirin, and a beta blocker and dissuaded from surgery due to its high risk.

Which of the following is indicated and may improve both survival and symptoms?

7. A 44-year-old man with an idiopathic dilated cardiomyopathy reports having become more sedentary, noting increasing dyspnea on exertion and fatigue with mild activity. Over the past 2 years you have up-titrated dosages of his carvedilol and enalapril while starting spironolactone. In the past 3 weeks, attempts to further increase his diuretics have resulted in significant increases in serum creatinine. On physical examination, his heart rate is 60 bpm, blood pressure is 94/70 mm Hg, and his respiratory rate is 16 bpm. His neck veins are flat, and his lungs are clear to auscultation bilaterally. He has a regular rhythm and a II/VI holosystolic murmur best appreciated in the fifth intercostal space, radiating to the axilla. An ECG reveals normal sinus rhythm with occasional ventricular premature contractions and a QRS duration of .13 sec. An echocardiogram performed in your office reveals moderate mitral regurgitation with moderate left atrial enlargement, global hypokinesis with marked left ventricular enlargement, and a LVEF of 30%. Right ventricular function is normal with a right ventricular systolic pressure of 33 mm Hg. His laboratory results include hemoglobin of 12 g/dL (120 g/L) and a creatinine of 1.1 mg/dL (97.2 μmol/L).

  1. The addition of digoxin.
  2. Mitral valve repair.
  3. Placement of a biventricular pacemaker and implantable cardiac defibrillator.
  4. Placement of an implantable cardiac defibrillator alone.
  5. The addition of erythropoietin.

:

8. All of the following are true of heparin except

  1. It inactivates thrombin.
  2. It inhibits factor VII.
  3. It inactivates activated factor X (Xa) through an antithrombin-dependent mechanism.
  4. It binds to antithrombin III to inactivate factor XIIa, factor XIa, and factor IXa.
  5. Thrombin and factor Xa are most sensitive to the effects of heparin/AT-III.

Based on large study populations, what therapy or therapies offers this patient the maximal benefit in survival?

9. A 59-year-old man with chronic stable angina for 4 years recently noticed neck and left arm pain while jogging that was relieved with rest. No other symptoms were reported. He is a former smoker. He has hyperlipidemia, hypertension, and insulin-dependent diabetes mellitus, all under excellent medical control. He is currently taking carvedilol 25 mg twice daily, lisinopril 10 mg each day, aspirin 325 mg each day, atorvastatin 80 mg a day, and insulin. Physical examination revealed only the presence of an S4. A resting ECG shows left ventricular hypertrophy and left atrial enlargement. An exercise nuclear stress test showed him exercising for 6 minutes on a Bruce protocol with anterior and apical ischemia. The anterior wall and apex were hypokinetic with an LVEF calculated at 38%. Cardiac catheterization reveals a 90% proximal stenosis in the LAD artery, an 85% stenosis in the proximal right coronary artery, and an 80% stenosis in the proximal left circumflex coronary artery.

  1. Coronary artery bypass surgery.
  2. Staged percutaneous coronary intervention (PCI).
  3. The addition of clopidogrel.
  4. The addition of ranolazine.
  5. All of the above.

Data that suggest a diagnosis of hypertrophic cardiomyopathy include all of the following except:

10. An 18-year-old woman is seen for her pre-college physical examination. She reports no symptoms or family history of cardiovascular disease. On physical examination, a II/VI crescendo-decrescendo systolic murmur is best heard in the fourth intercostal space, left sternal border.

  1. Lack of radiation to the carotid arteries.
  2. An increase in intensity with the Valsalva maneuver.
  3. An increase in intensity from the squatting-to-standing position.
  4. An increase in intensity during passive leg elevation.
  5. A decrease in intensity during isometric handgrip.

»Click to view answers

Related Videos
Matthew Nudy, MD | Credit: Penn State Health
Kelley Branch, MD, MSc | Credit: University of Washington Medicine
Kelley Branch, MD, MS | Credit: University of Washington Medicine
David Berg, MD, MPH | Credit: Brigham and Women's
HCPLive Five at ACC 2024 | Image Credit: HCPLive
Ankeet Bhatt, MD, MBA | Credit: X.com
Ankeet Bhatt, MD, MBA | Credit: X.com
Sara Saberi, MD | Credit: University of Michigan
Muthiah Vaduganathan, MD, MPH | Credit: Brigham and Women's Hospital
Albert Foa, MD, PhD | Credit: HCPLive
© 2024 MJH Life Sciences

All rights reserved.